User contributions
(newest | oldest) View (newer 50 | older 50) (20 | 50 | 100 | 250 | 500)
- 20:06, 17 November 2022 (diff | hist) . . (+817) . . 2022 AMC 10B Problems/Problem 13 (→Solution)
- 19:49, 17 November 2022 (diff | hist) . . (+595) . . 2022 AMC 10B Problems/Problem 13 (→Solution)
- 18:26, 17 November 2022 (diff | hist) . . (+138) . . N 2022 AMC 10B Problems/Problem 13 (Created page with "==Solution== Let the two primes be <math>a</math> and <math>b</math>. We would have <math>a-b=2</math> and <math>a^{3}-b^{3}=31106</math>")
(newest | oldest) View (newer 50 | older 50) (20 | 50 | 100 | 250 | 500)